Problem

Source: USA TSTST 2018 Problem 4

Tags: algebra



For an integer $n > 0$, denote by $\mathcal F(n)$ the set of integers $m > 0$ for which the polynomial $p(x) = x^2 + mx + n$ has an integer root. Let $S$ denote the set of integers $n > 0$ for which $\mathcal F(n)$ contains two consecutive integers. Show that $S$ is infinite but \[ \sum_{n \in S} \frac 1n \le 1. \] Prove that there are infinitely many positive integers $n$ such that $\mathcal F(n)$ contains three consecutive integers. Ivan Borsenco